11. Determine the type and number of solutions of x2 - 8x + 16 = 0.
O two imaginary solutions
one real solution and one imaginary solution
O one real solution
O two real solutions

Answers

Answer 1

Answer:

one real solution

Step-by-step explanation:

the equation simplified is (x+4)²

Answer 2

Option 3 (two real solutions) is the right answer.

What is a Quadratic equation and its solution?

A quadratic equation in the variable x is an polynomial of degree 2 equation of the form ax²+bx+c = 0 where a, b, c are real numbers a≠0. For example, 2x²-3x+1 =0. And we say that x=α is an solution of a quadratic equation if α satisfies the equation and since its degree is two it will always have 2 solutions which may be real or Imaginary.

Given equation here is : x² -8x+16= 0

                                       x²-2.4x+4²=0

                                      (x-4)²=0

                                     x=4,4

Hence, the quadratic equation has two equal and real solutions.

Learn more about quadratic equation here:

https://brainly.com/question/17177510

#SPJ5


Related Questions

Question #19- Use the answer bank below to fill in missing parts of the proof.
Given: PQPR
Prove: QR
P
R

Answers

Answer:

∠MPQ ≅ ∠MPR: Reason; Corresponding parts of congruent triangles are congruent (CPCTC)

∠PQR ≅∠PRQ: Reason; CPCTC

Step-by-step explanation:

[tex]\overline{PQ}\cong \overline{PR}[/tex]: Reason; Given

Draw [tex]\overline{PM}[/tex] so that M is the midpoint of [tex]\overline{QR}[/tex]: Reason; Two points determine a line

[tex]\overline{QM}\cong \overline{RM}[/tex]: Reason; Definition of midpoint

[tex]\overline{PM}\cong \overline{PM}[/tex]: Reason; Reflexive property

ΔPQM ≅ ΔPRM: Reason; Side Side Side (SSS) rule for triangle congruency

∠MPQ ≅ ∠MPR: Corresponding parts of congruent triangles are congruent CPCTC

∠PQR ≅∠PRQ: CPCTC

Using the following image, apply what you have learned about linear pairs and solve for

Answers

It would be 4x+76 = 180 in that order in the boxes. You would add 1 to 75 which got 76. Then you would subtract 76 from 180 and get 104 and then divide that by 4 and you would get 26. So x is 26

Find the value of x in the question

Answers

Answer:

x=52

Step-by-step explanation:

r=90 s=34  q=180-(90+34)

180-(90+34)=56

to find angle sqr

180-56=124

124=x+72

52=x

Answer:

x = 52

Step-by-step explanation:

→We are given an exterior angle (outside of the triangle) as (x + 72). We are also given two interior angles as 34 and 90. The sum of the two farthest interior angles away from the exterior, is equal to the exterior. So this means that ∠R(90°) + ∠S(34°) = ∠PQS(x + 72), and we can set up the equation like that:

90 + 34 = x + 72

→Add like terms (90 and 34):

124 = x + 72

→Subtract 72 from both sides:

52 = x

if you translate the cubic parent function, F(x) down 2 units, what is the equation of the new parent function

Answers

Answer:

y=f(x)-2

Step-by-step explanation:

lcm of 6 and 13 what is it?

Answers

Answer:

78

Step-by-step explanation:

Since 13 is a prime, the least common multiple of it and 6 is their product. 6*13=78. Hope this helps!

Answer:

78

Step-by-step explanation:

Least Common Multiple of 6 and 13. Least common multiple (LCM) of 6 and 13 is 78.

6- 6, 12, 18, 24, 30, 36, 42, 48, 54, 60, 66, 72, 78

13- 13, 26, 39, 52, 65, 78

sarah wants to open up her own coffee shop. It costs her $3 to make each cup of coffee, plus she has to pay a $24 renter's fee for her shop. She sells the coffee for $5 per cup. How many cups of coffee does she has to sell before she can start making a profit? *

Answers

Answer:

5

Step-by-step explanation:

So you divide 24 by 5 and you get 4.8 so you round it up to 5

Write the slope of DB

Answers

the slope is c/b-a i’m pretty sure

Answer: [tex]\frac{c - 0}{b-a}[/tex]

Which of these is equal to 0.13?
13
A.
99
12
B.
90
c. 1

Answers

Answer:

Is this the inter question

For which set of data will the scatter plot represent a positive linear association between x and y?
O Set A
O Set B
O Setc
O Set D
PLEASE HELP ME

Answers

Answer:

Set C

Step-by-step explanation:

Answer:

The answer is Set B

Step-by-step explanation:  

Positive linear association goes from left to right and in a straight line. If you put the information into a scatter plot, the line moves in the positive direction and is in a straight line. (In a scatter plot, x goes across the bottom and y is vertical.)

Somebody please help me. I cant figure this out once so ever.

Answers

Answer:

A

Step-by-step explanation:

The Fibonacci sequence is the sum of the two terms before it. It looks like this:

1, 1, 2, 3, 5, 8, 13, 21, 34, ...

As you can see, 13 and 21 are indeed consecutive terms.

What is the value of this expression when c= -4 and d = 10?
(c+d2)
O A.
2.
OB.
9
Ос.
21
OD. 41

Answers

Answer:

[tex]c + d^2 = 96[/tex]

Step-by-step explanation:

Given

[tex]c = -4\\d = 10[/tex]

Required

Find [tex]c + d^2[/tex]

To find [tex]c + d^2[/tex], we simply substitute 10 for d and -4 for c in the given expression.

The step is as follows:

[tex]c + d^2[/tex]

Substitute [tex]c = -4\\d = 10[/tex]

[tex]c + d^2 = -4 + 10^2[/tex]

[tex]c + d^2 = -4 + 10*10[/tex]

[tex]c + d^2 = -4 + 100[/tex]

[tex]c + d^2 = 96[/tex]

The expression cannot be solved any further; so, we can conclude that when d = 10 and c = -4, [tex]c + d^2 = 96[/tex]

What has to be added to the sum of 4/7 and 7/4 to get 3?

Answers

Answer:

if you add 1 u get 3

Step-by-step explanation

Answer:

[tex]0.67858[/tex]

Step-by-step explanation:

[tex]\frac{4}{7}+\frac{7}{4}\\\mathrm{For}\:\frac{4}{7}:\:\mathrm{multiply\:the\:denominator\:and\:numerator\:by\:}\:4\\\frac{4}{7}=\frac{4\cdot \:4}{7\cdot \:4}=\frac{16}{28}\\\mathrm{For}\:\frac{7}{4}:\:\mathrm{multiply\:the\:denominator\:and\:numerator\:by\:}\:7\\\frac{7}{4}=\frac{7\cdot \:7}{4\cdot \:7}=\frac{49}{28}\\=\frac{16}{28}+\frac{49}{28}\\\mathrm{Since\:the\:denominators\:are\:equal,\:combine\:the\:fractions}:\quad \frac{a}{c}\pm \frac{b}{c}=\frac{a\pm \:b}{c}\\=\frac{16+49}{28}[/tex]

[tex]\mathrm{Add\:the\:numbers:}\:16+49=65\\=\frac{65}{28}\\\mathrm{Decimal:\quad }\:2.32142\\3-2.32142=0.67858[/tex]

50 POINTS IF YOU GET THIS QUESTION!!! Find area of the following shape if the vertices are X=(1,1), Y=(2,5), Z=(5,5), W=(8,1)

Answers

Answer:

20

Step-by-step explanation:

To find area using coordinates, you would take the y-value of the first coordinate and the x-value of the second coordinate and multiply them. Then, take the x-value of the first coordinate and multiply it with the second y-coordinate. Take the second number and subtract it from the first number. for multiple coordinates, you would multiply the first and the second, the second and the third, and so on. Once you reach the last coordinate, just multiply it by the first again. then, take all of the numbers, from multiplying your coordinates and add them together and divide them by 2 to find your area.

We would start with coordinate (8,1) here because it's letter variable is W, which is first of the others alphabetically.

(8,1) (1,1)

1 x 1 = 1

8 x 1 = 8

1 - 8 = -7

next coordinate:

(1,1) (2,5)

1 x 2 = 2

1 x 5 = 5

2 - 5 = -3

next:

(2,5) (5,5)

5 x 5 = 25

2 x 5 = 10

25 - 10 = 15

next:

(5,5) (8,1)

5 x 5 = 40

5 x 1 = 5

40 - 5 = 35

-7 - 3 + 15 + 35 = 40

40/2 = 20

Answer:

[tex]\boxed{\mathrm{view \: attachment}}[/tex]

Step-by-step explanation:

Which equations and/or functions represent the graphed line? Select three options.
4
.
..6
Х
1

Answers

Answer:

f(x) = 0.5x + 2

f(x) = 1/2x + 2

y - 3 = 1/2(x - 2)

y - 1 = 0.5(x + 2)

Step-by-step explanation:

In the figure attached, the graphed line is shown.

The missing options are:

f(x) = 0.2x - 4

f(x) = 0.5x + 2  

f(x) = 1/2x + 2

y – 3 = 1/2(x – 2)

y – 1 = 0.5(x + 2)

From the picture, we can see that points (0, 2) and (2, 3) are on the line. Then, the slope of the line is:

m = (3 - 2)/(2 - 0) = 1/2 = 0.5

The y-intercept is (0, 2), or b = 2

Therefore, in the slope y-intercept form, the equation is:

f(x) = mx + b

f(x) = 1/2x + 2 = 0.5x + 2

In the point-slope form, the equations is:

y - y1 = m(x - x1)

y - 3 = 1/2(x - 2)

Using point (-2, 1), in the point-slope form, the equation is:

y - y1 = m(x - x1)

y - 1 = 0.5(x + 2)

Answer:

bde

Step-by-step explanation:

-7 -5(-4c+4) <-7c+8-3

Answers

Answer:

c  <  32/27

Step-by-step explanation:

−7−5(−4c+4)<−7c+8−3

Step 1: Simplify both sides of the inequality.

20c−27<−7c+5

Step 2: Add 7c to both sides.

20c−27+7c<−7c+5+7c

27c−27<5

Step 3: Add 27 to both sides.

27c−27+27<5+27

27c<32

Step 4: Divide both sides by 27.

27c/27  <  32/27

c <  32/27

the answer is c < 32/27

Let M(x)=4x2−3x and N(x)=−5x3−6x2−3. Find M(x)−N(x)

Answers

Answer:

5x³+10x²-3x+3

Step-by-step explanation:

(4x²-3x) - (-5x³-6x²-3)

5x³+10x²-3x+3

If equations M(x)=[tex]4x^{2} -3x[/tex] and N(x)=[tex]-5x^{3}-6x^{2} -3[/tex] then M(x)-N(x)=[tex]5x^{3} +10x^{2} -3x+3[/tex].

What is equation?

An equation is a relationship between two or more variables expressed in equal to form. It is often equated to find the values of variables. The form of an equation is ax+by=c.

How solve equations?

We have been given M(x)=[tex]4x^{2} -3x[/tex] and N(x)=[tex]-5x^{3}-6x^{2} -3[/tex] and we have to find the difference between M(x) and N(x) and to solve them we have to open the brackets and add the coefficients of variable.

M(x)-N(x)=[tex]4x^{2} -3x-(-5x^{3}-6x^{2} -3)[/tex]

First we have to open the brackets.

=[tex]4x^{2} -3x+5x^{3}+6x^{2} +3[/tex]

Now we have to add the coefficients of same variable having same powers.

[tex]=5x^{3}+10x^{2} -3x+3[/tex]

Hence if M(x)=[tex]4x^{2} -3x[/tex] , N(x)=[tex]-5x^{3}-6x^{2} -3[/tex] then difference between {M(x)-N(x)} =[tex]5x^{3}+10x^{2} -3x+3[/tex].

Learn more about equations at https://brainly.com/question/2972832

#SPJ2

yall i really need help on this one :/ Mrs. Berry asked her real-estate agent, "What is the median value of homes in this neighborhood?"
Select from the drop-down menus to correctly complete the statement.

Mrs. Berry's question is a statistical question because there ____ variability in the values, and because data _____ to be collected to answer it.
for the first blank the options are is and is no. for the second blank the options are has and do not have.

Answers

non stat,does not varyStep-by-step explanation:


Definition of Complementary Angles

Answers

either of 2 angles whos sum is 90°

Answer:

Add to 90 degrees

Step-by-step explanation:

Complementary angles are angles that add up to 90 degrees or a right angle. For example,

52°+38°=90°

Therefore, 52 and 38 are complementary angles.

If m∠9 ≅ 95°, find m∠14. answers: Question 5 options: 85° 95° 90° 88°

Answers

Answer:

Option (1). 85°

Step-by-step explanation:

From the figure attached,

'l' and 'm' are the parallel lines and line 'n' is a transverse.

Since, m∠9 ≅ m∠13 [These angles are the corresponding angles]

Therefore, m∠13 = 95°

Since, m∠13 + m∠14 = 180° [Supplementary angles]

Therefore, 95° + m∠14 = 180°

m∠14 = 180° - 95°

m∠14 = 85°

Therefore, 85° will be the measure of angle 14.

Option (1) will be the answer.

PLEASE HELP!! The question is “ Abbey spends 5 hours less then triple the amount of time Emily spends on homework. Nathan spends twice as much time on homework as Abbey does. Write a simplified expression for the total number of hours the three friends spend on their homework. Show your work.”

Answers

Answer:

5(2x-3)

Step-by-step explanation:

Let the amount of time Emily spends on homework.=x (in hours)

Abbey's Time = 5 hours less than triple the amount of time Emily spends

Therefore:

Amount of Time spent by Abbey =3x-5

Nathan's Time = Twice as much time as Abbey.

Amount of Time spent by Nathan =2(3x-5)

Next, we calculate the total number of hours the three friends spend on their homework

=Time spent by Emily+Time spent by Abbey+Time spent by Nathan

=x+(3x-5)+2(3x-5)

=x+3x-5+6x-10

Collect like terms

=x+3x+6x-5-10

=10x-15

Next, we simplify by factorizing

Therefore, Total number of hours =5(2x-3)

Board A: 3ft 9in Board B: 4ft 3in Board C: 2ft 6in
If he glues boards A and C together end to end, how long will the new board be?

Answers

The board will be 75 inches long

Choose the point on the terminal side of -45°.

a. (-3, -3)
b. (4, -4)
c. (5, 5)
d. (-2, 2)

Answers

Answer:

B

Step-by-step explanation:

Which two numbers doesStartRoot 61 EndRoot lie between on a number line? 7.7 and 7.8 7.8 and 7.9 7.9 and 8.0 8.0 and 8.1

Answers

Answer:

Answer is 7.8 and 7.9

Step-by-step explanation:

Answer:

the answer is B 7.8 to 7.9 have fun

Step-by-step explanation:

Casey has a litter of 5 puppies and 3 of them are gray. At this rate, how many puppies would be gray in a litter of 30 puppies?

Answers

Answer:

18 gray puppies

Step-by-step explanation:

Think of this as finding equivalent fractions/ratios. 3/5 puppies are gray. what over 30 puppies would be gray? to find 3/5=?/30, you multiply the numerator and denominator by 6. 3 times 6 is 18.

Find the median of both you and your friend’s basketball scores. Who’s score is greater? By how much?

Answers

Answer:

Find the number in the middle

First list the number from smallest to greatest

6,8,10,10,12,14,14,16,16,18,18,20,20,22

Find which one is in the middle

16 and 16 are the two middle numbers. There is two middle numbers because there is an even amount of numbers

If there are two middle numbers find the average

Do (X+Y) / 2

16+16 = 32

32 / 2 = 16

16 is the median for your score

Do the same to your friend's score

2,8,8,10,10,10,12,12,12,12,14,14,14,16,16

12 is the middle because there is 7 numbers to the left and right so 12 is the median.

Your score is greater by 4 because 16 - 12 = 4.

Hope this helps

Step-by-step explanation:

The rectangular prisms below have the same size and shape.

What is the combined volume, in cubic inches, of the prisms?

A: 8

B: 16

C: 32

D:64

Answers

A is the answer and all u have to do is nothing
it’s A hoped I helped

the cougars baseball team played 16 games last year and won 12 of them. this year they are scheduled to play 28 games. if the end up with the same winning percentage, how many games will they win

Answers

Answer:

They won 21 games

Step-by-step explanation:

First take the 12 out of 16 games they have won and make it a fraction, 12/16. Then divide 12 by 16, 12 ÷ 16 and then you get, 0.75 then as always multiply it by 100 and get 75 (you can also just move the decimal over 2 places) Then after that you smack that percentage sign on 75 and get %75. So now we know that the Cougars baseball team won %75 of there games. Now take 28 and put it as a fraction, but! You need two numbers for it to be a proper fraction to we will replace the numerator with W to represent how many games they won, so now you should have something like this W/28, so now we find the value of W and by doing that just guess and check (it cant be over 28 so it must be under 28) and when your looking for your number do W ÷ 28 and try to get 0.75 and when you get that (that being 21 ÷ 28 = 0.75 ) you now know that 21 is your answer.

Hope this helps but im in 6th so..... dont quote me on this some of the explanation might not be clear to you but it is to me hope you can figure it out

Consider the statement P: x + 9 = 10. Which of the following is an equivalent statement? Question 13 options: A) R: x > 0 B) Q: x – 9 = 10 C) S: x = 1 D) ∼P: x + 9 ≠ 10

Answers

Answer:

C) S: x = 1

Step-by-step explanation:

Answer:

C) S: x = 1

explination

Polygons QRST and Q’R’S’T’ are shown on the coordinate grid: A coordinate plane with two polygons is shown. Polygon QRST has vertices Q at 3 comma negative 5, R at 2 comma negative 1, S at 5 comma 0, and T at 5 comma negative 4. Polygon Q prime R prime S prime T prime has vertices at Q prime negative 5 comma negative 4, R prime at negative 1 comma negative 3, S prime at 0 comma negative 6, and T prime at negative 4 comma negative 6. What set of transformations is performed on QRST to form Q’R’S’T’? A 180-degree clockwise rotation about the origin followed by a translation 1 unit to the right A 180-degree clockwise rotation about the origin followed by a translation 1 unit to the left A translation 1 unit to the left followed by a 270-degree counterclockwise rotation about the origin A translation 1 unit to the right followed by a 270-degree counterclockwise rotation about the origin

Answers

Answer:

the answer is D

Step-by-step explanation:

If you take wha it says and plug it in, it makes the most sense

Answer:

d

Step-by-step explanation:

(4a^4)^2 write without exponents

Answers

(4 x a x a x a x a) x (4 x a x a x a x a)

Answer:

16 a^8 = 16aaaaaaaa

Step-by-step explanation:

(4a^4)^2

We know (xy)^z = x^z y^z

4^2 a^4^2

16 a^4^2

We know a^b^c = a^(b*c)

16 a^(4*2)

16 a^8

Other Questions
Whats the correct answer for this? Combine the following sentences using appropriate conjunctions to have just one sentence. 2.a) One effect of studying abroad is a students greater understanding. b) The effect is important. c) The understanding is of an educational system. d) The system is different Rectangle ABCD is graphed in the coordinate plane. The following are the vertices of the rectangle: A(3,2), B(6, -2), C(6, 5) D(3,5). What is the area of this rectangle What is one thing you could do today to help you and our friends delete digital drama? Which journal entry reflects the adjusting entry needed on December 31?:In November, BOC received a $5,000 cash deposit from a customer for custom-build goods that will be delivered in January (BOC recorded an entry for this $5,000 in November). Now, it is December 31, the end of the fiscal year.Dr. Unearned Revenue 5,000Cr. Inventory 5,000No entry needed.Dr. Cash 5,000Cr. Revenue 5,000Dr. Advances from Customers 5,000Cr. Revenue 5,000Dr. Unearned Revenue 5,000Cr. Revenue 5,000 do what you gotta do. helppp meh What is the perimeter of triangle PQR? How did the Middle Ages fall due to the feudal system? An open window is at the height of 15 ft above the ground. A ladder is placed 8 ft away from the side of the house. What will be the length of ladder needed to reach to the window of the house? Describe fully the single transformation that maps triangle a onto triangle b A rectangular swimming pool is 17 meters long, 13 1/2 meters wide, and 2 1/2 meters deep. What is its volume? need help!! plzz help asap A process is in control with mean 50 and standard deviation of 2. The upper specification limit for the product being produced is 60 and the lower specification limit is 42. What is the value of Cpk? If you roll two fair dice how many different ways can you obtain a different number on each side PLEASE HELP WILL MARK BRAINLIEST !!!!!!! 28 ptsThe price of the box of 10 markers is $7. The price of the box of 24 markers is $14. All prices without tax and the price of the boxes are the same.Part A:Write an equation that tells the price Y for the number of markers X.Part B:How much would a box of 64 markers cost? Calculate the empirical formula of a compound with the followingpercentage composition: 31,8% K; 29,0% C1; 39,2% OThe empirical formula of ethane is CH. Its molar mass is 30 g.mol- Which statement Best describes the effect of tone on the reader?A. The tone helps the reader learn details about the authors background.B. The tone helps the reader make an informed decision about a subject.C. The tone helps the reader interpret the structural plot of a text.D. The tone helps the reader interpret an authors attitude on a subject.Reset Next What is the Slope of the line through (-1,2) & (-3,-2) I promise this isn't a test .-. PQ has endpoints P(6,-3) and Q(-2,-7). Find the coordinates of the midpoint of PQ. When will a real dog cost more than an Aibo?